Reduction of Order Method - Differential Equation Help

Click For Summary
The discussion focuses on solving two differential equations using the reduction of order method, specifically avoiding the y2 formula. The first problem involves the equation (1 - x^2)y'' + 2xy' = 0 with a known solution y1 = 1, while the second problem is 4x^2y'' + y = 0 with y1 = [x^(1/2)]ln|x|. Participants suggest transforming the equations by letting w = y' for the first problem to simplify it into a first-order differential equation. For the second problem, it is recommended to express y as v*y1 to find another solution. The thread emphasizes the need for clearer instructional guidance on these methods.
bathtub2007
Messages
2
Reaction score
0
1. Problem Statement

Okay so I know these appear to be simple but for some reason I am having trouble finding the methods by which to solve them.

Problem 1:

This problem must be solved by reduction of order method and cannot use the y2 formula.

(1 - x^2)y'' + 2xy' = 0; y1 = 1


Problem 2:

This problem must be solved by reduction of order method and cannot use the y2 formula.

4x^2y'' + y = 0; y1 = [x^(1/2)]ln|x|



Homework Equations



y2 / y1 = u (x)

y2 = u (x) * y1 (x)

y' = u'x + u

y'' = u''x +2u'


The Attempt at a Solution



The general form for the problem is:

eq0001M.gif



I have been trying to use Paul's Notes (http://tutorial.math.lamar.edu/Classes/DE/ReductionofOrder.aspx) to help me along the way but I am unable to follow for the lack of the teacher not exactly teaching the problems.


Any help getting towards the right direction would be much appreciated. Thank you.
 
Last edited by a moderator:
Physics news on Phys.org
For the first one...try putting w=y' such that w'=y''

Now you have a first order DE.

For the second one, if y=y1 is a solution, then y=v*y1 is another solution.
 
Question: A clock's minute hand has length 4 and its hour hand has length 3. What is the distance between the tips at the moment when it is increasing most rapidly?(Putnam Exam Question) Answer: Making assumption that both the hands moves at constant angular velocities, the answer is ## \sqrt{7} .## But don't you think this assumption is somewhat doubtful and wrong?

Similar threads

  • · Replies 2 ·
Replies
2
Views
1K
  • · Replies 7 ·
Replies
7
Views
2K
Replies
7
Views
2K
Replies
10
Views
2K
  • · Replies 7 ·
Replies
7
Views
2K
  • · Replies 2 ·
Replies
2
Views
2K
  • · Replies 1 ·
Replies
1
Views
1K
  • · Replies 3 ·
Replies
3
Views
909
  • · Replies 1 ·
Replies
1
Views
1K
Replies
1
Views
1K